Holistic Numerical Methods

Transforming Numerical Methods Education for the STEM Undergraduate

 

MOOC | MOBILE | VIDEOS | BLOG | YOUTUBE | TWITTER | COMMENTS | ANALYTICS | ABOUT | CONTACT | COURSE WEBSITES | BOOKS | MATH FOR COLLEGE


NUMERICAL METHODS BOOK ERRATA (Book Bought BETWEEN 1/1/2013- NOW)

Page 5 of Chapter 01.04, a student pointed out a typo - it should be ai=S in one of the blocks.  It has been corrected here: http://mathforcollege.com/nm/mws/gen/01aae/mws_gen_aae_spe_binaryrepresentation.pdf

 

Page 3 of Chapter 04.07. CT|invLU = 1*CT|LU DE+ n*CT|FS + n*CT|BS.  It has been corrected at http://mathforcollege.com/nm/mws/gen/04sle/mws_gen_sle_txt_ludecomp.pdf

 

Page 15 of Chapter 04.06: On top of the page, the right hand side vector has 7 for the first row. It should be 45.  It has been corrected at http://mathforcollege.com/nm/mws/gen/04sle/mws_gen_sle_txt_gaussian.pdf

 

NUMERICAL METHODS BOOK ERRATA (Book Bought BETWEEN 12/24/2009- 12/12/2012)

Page 566 (last line of page, that is Eqn 9) should read as

                           

 

Page 333 (Under Figure 4 caption)
"Since we want to find the velocity at t = 16 , and we are using a third second order polynomial"

 


If you find any errors in the book, please email them to autarkaw@yahoo.com or anonymously fill this web form.  If you are at USF, just see me in class or office or slide it under my office door or leave a message at 813-974-5626.  You do not have to mention the error completely as it may involve equations, but a page number and line number would be helpful.  Thank you!


NUMERICAL METHODS BOOK ERRATA (Book Bought BETWEEN 04/25/2009 -12/23/2009)

Page 15: Line 19 from bottom should read as


NUMERICAL METHODS BOOK ERRATA (Book Bought BEFORE 04/25/2009)


Page 5, second line from top:  0.013783    - 0.013783 (put the minus sign in front of 0.013783)

_______________________________________________________________

Page 15, 4th line from bottom: 0.5x102-m  0.5x102-m%


Page 27, the correct answer for Q4 is A

Page 27, the choices for Q5 need to be (A) 648 (B) 756 (C) 972 (D) 1620.  The correct answer is C.

Page 28, the choices for Q6 need to be (A) 0.082 (B) 135 (C) 270 (D) 5400.

For the above three corrections, you can also replace page 27-28 by this revised version

____________________________________________________________________

Page 45: Replace Page 45 with page 3 of this document.  Example 1 has some mistakes and is not posed completely.

Page 48: In Example 4, the reference to Example 2 should be Example 3.

Page 67: In Problem 4, the series should have (-1)n rather than just (-1).

Page 160: 3rd last line: It should read as

_____________________

Page 465:

should read as

 

Page 531 Q5: The choices need to be

(A)  2500 m2

(B)  4775 m2

(C)  5250 m2

(D)  6000 m2

Page 592: The lower limit of integration is 2.

Page 623:  The correct answer to #5 Page 623 is (B)

Page 633: Line 9 from bottom should read as

Page 680: Line 13 from top should read as: From Equation (E3.4b)

Page 685: The ODE is